Gibt es eine vierte Komponente zum elektrischen Feld und zum magnetischen Feld?

Die Frage

Wenn die drei elektrischen und magnetischen Vektorfelder aus dem vierkomponentigen Vierpotential stammen, gibt es dann eine vierte Komponente für das elektrische und magnetische Feld?

Verwandte Frage

Ich habe die folgende Frage gepostet: Was ist die physikalische Bedeutung der Dipoltransformation der Maxwell-Gleichungen? . Ich erinnere mich, dass ich vor einiger Zeit Maxwells Gleichungen aufgeschrieben und die Felder mit dem Positionsvektor gekreuzt habe und ich in der Lage war, Maxwells Gleichungen von einer Monopol-Ladungsquelle in etwas umzuwandeln, das wie eine Feldgleichung mit Dipolquelle aussah. Als ich dies tat, enthüllte es eine bizarre vierte Komponente in den Feldgleichungen, die meiner Meinung nach mit dieser Frage zusammenhängen könnte,

Antworten (6)

Tatsächlich bilden die elektrischen und magnetischen Felder einen kombinierten Tensor, der elektromagnetischer Feldtensor genannt wird . Dies ist ein Rang-2-Tensor und hat die Form *

F μ v = ( 0 E x E j E z E x 0 B z B j E j B z 0 B x E z B j B x 0 )
Es hat die folgenden Eigenschaften:

  1. Es ist antisymmetrisch (also F 12 = F 21 )
  2. Es ist spurlos
  3. Es hat 16 Elemente, aber nur 6 verschiedene Werte
  4. Bei Multiplikation mit seinem dualen Tensor ( G μ v ) gibt es einen Lorentz-Invariantenwert von 4 B E
  5. Das innere Produkt, F μ v F μ v = 2 ( B 2 E 2 ) , ist ebenfalls eine Lorentz-Invariante

Sie können die Maxwell-Gleichungen auch durch den Tensor durch Anwendung ableiten μ dazu. Gaußsches Gesetz und Amperesches Gesetz stammen

μ F μ v = 4 π J v
wo J μ = ( ρ , j ) ist der Vierstrom. Das magnetische Gaußsche Gesetz und das Faradaysche Gesetz ergeben sich aus der Anwendung der Bianchi-Identität , um zu erhalten
γ F μ v + μ F v γ + v F γ μ = 0
Oder kürzer,
[ μ F v γ ] = 0


Ich bin Astrophysiker, also verwende ich cgs-Einheiten; in SI haben alle elektrischen Felder einen Faktor von 1 / c .

Um diese Frage zu beantworten, müssen Sie ein vollständiges geometrisches Verständnis der Maxwell-Gleichungen und ihrer Bedeutung haben.

Die Maxwell-Gleichungen sind ein artenreiches System von PDEs. In der STA-Notation ist es einfach

F = J

Das halten wir für selbstverständlich F ist ein Bivektor, hat also 6 Komponenten, und das J ist ein Vektor, hat also 4 Komponenten. Aber diese Gleichung beschreibt bis zu acht separate Gleichungen. Warum ist das so?

Für ein beliebiges Bivektorfeld K , Die Ableitung K kann sowohl Vektor- als auch Trivektorterme haben. Dass die Maxwell-Gleichungen nur einen vektoriellen Quellterm haben, ist eigentlich ziemlich bedeutsam: Das ist Teil des physikalischen Inhalts der Maxwell-Gleichungen. Wir sagen, das EM-Feld wird nur durch einen Vektorstrom bestimmt.

Was würde passieren, wenn es einen Trivektor-Stromquellenterm gäbe? Es wäre eine "magnetische" Ladung (magnetische Monopole) und ein damit verbundener Strom. Wir können also sofort erkennen, was dieser Quellbegriff bedeuten würde.

Aber warte, es gibt noch mehr! Nehmen wir an, wir hatten damals sowohl elektrische als auch magnetische Ströme. Welche Arten von Feldern könnten sie hervorbringen?

Wie Sie zu verstehen versucht haben, sind dies die anderen beiden Komponenten eines Feldes, die in diese Differentialgleichung eingehen könnten. Sie sind ein skalares Feld und ein pseudoskalares Feld. Ich bin nicht damit vertraut, wie sich diese Felder manifestieren oder was sie tun würden.


Warum finden wir es nicht heraus?

Lassen λ sei das Skalarfeld. Wie würde sich dies auf die Maxwell-Gleichungen mit nur einem aktuellen Quellterm auswirken?

Lassen F = e 0 E + B , wobei ich implizit angegeben habe, dass das Magnetfeld ein Bivektor ist . Sie können es stattdessen als Vektor identifizieren und dann überlegen ϵ 3 B , aber der Nettoeffekt ist ziemlich minimal.

Die Maxwell-Gleichungen brechen dann zusammen als

F = t E e 0 3 E + 3 B = ρ e 0 j

und

F = e 0 3 E e 0 t B + 3 B = 0

Skalarfeld hinzufügen λ würde nur den Vektorteil mit seinem Gradienten betreffen:

J = F e 0 t λ + 3 λ

Alles in allem würde dies wahrscheinlich als eine Art zusätzlicher Strom erscheinen, der nicht mit den Bewegungen elektrischer Ladungen verbunden ist – oder vielleicht wäre er in irgendeiner Weise nicht von elektrischen Strömen zu unterscheiden, außer dass er den gesamten Raum als kontinuierliche Funktion durchdringt. Es sieht so aus, als gäbe es gewissermaßen überall Strömungen. Sie können sehen, warum wir die Existenz eines solchen Feldes nicht einmal in Betracht ziehen. Wenn es nicht sehr klein ist, hätten wir es schon vor einiger Zeit entdeckt, da es mit dem Begriff der elektrischen Stromquelle interagiert.

Eine Analyse des magnetischen pseudoskalaren Feldes würde wahrscheinlich genauso enden.


Fehlen dem Faraday-Tensor also tatsächlich zwei zusätzliche Komponenten, ein skalares Feld und ein pseudoskalares Feld? Ich würde nein sagen , aber wenn Sie etwas anderes entdecken, werden Sie wahrscheinlich einen Nobelpreis gewinnen. Viel Glück damit. Wie ich in der anderen Frage sagte, lassen Sie sich nicht täuschen , nur weil das so ist F x acht Komponenten hat, fehlen Komponenten des Faraday-Feldes. Es gibt sehr wahrscheinlich keine derartigen fehlenden Komponenten. Sie können dies sehen, indem Sie überlegen, was diese Komponenten in den Vanilla-Maxwell-Gleichungen tun würden, wie ich es hier getan habe.


Bearbeiten: Einige Korrekturen zur Beziehung zwischen diesem Skalarfeld und der Messgerätfixierung.

Dieses skalare Feld würde die Freiheit zur Veränderung aufheben EIN durch Eichtransformationen, wie λ würde die Abweichung von angeben EIN . Erinnern Sie sich, dass die Fixierung des Messgeräts von der Fähigkeit abhängt, die Transformation durchzuführen,

EIN EIN + χ

Für ein Skalarfeld χ . Dies ist möglich, weil ( EIN + χ ) = EIN = F , also bleibt das EM-Feld unverändert.

Doch wenn EIN = λ , dann würde das Hinzufügen des Gradienten eines Skalarfelds den Wert von ändern λ messbar, in allen außer den einfachsten Fällen:

( EIN + χ ) = λ + 2 χ

Jetzt wären Sie auf Messfunktionen beschränkt χ die streng harmonisch sind. Harmonische Felder sind normalerweise solche, die aus einer Auswahl von Randbedingungen entstehen – dh dies würde einer Auswahl von Randbedingungen entsprechen, und der Feldbeitrag von Strömen wäre unverändert. Natürlich überfordert es die Vorstellungskraft, darüber nachzudenken, wie man dies vernünftigerweise für die Messgerätefixierung tun würde. Und wenn Sie eine Transformation gefunden haben, die bewahrt λ , es würde nicht gehen F allgemein unveränderlich.

Also die angebliche Existenz dieser Funktion λ tiefgreifende Konsequenzen für die Festsetzung der Spurweite haben würde. Es verbietet es nicht absolut, wie ich ursprünglich dachte, aber es schränkt die Reparatur ernsthaft ein, auf die wir wahrscheinlich schon gestoßen wären.

Vielen Dank für die hervorragende Antwort. Es hört sich so an, als würde dieser zusätzliche skalare Term, der im elektromagnetischen Feld auftreten könnte, die Ladungserhaltung aufheben? Ist es möglich, dass dieser skalare Term in Bezug auf die vier Potentiale beschrieben werden könnte?
+1. Diese Antwort bringt mich dazu, mehr Zeit mit dem Studium der geometrischen Algebra zu verbringen.
@linuxfreebird: Ja, dieses Skalarfeld würde vollständig spezifiziert EIN , und dann durch das Minkowski-Analogon des Helmholtz-Theorems, das spezifizieren würde EIN bis hin zu Randbedingungen. Die Existenz eines solchen Skalarfeldes würde sich darin manifestieren , dass die Fähigkeit zur Eichfixierung unterbrochen wird . Zum Beispiel könnten Sie normalerweise die Lorenz-Lehre durch Einstellung verwenden ( EIN ) = 0 . Aber hier, λ = EIN , und wenn λ existieren würde, hätten Sie a priori keinen Grund, seinen Gradienten (und damit seine zugehörigen aktuellen Terme) auf Null zu setzen.
Ich sollte sagen λ angeben würde EIN , und das zusätzlich zu F = EIN vollständig spezifizieren würde EIN bis hin zu Randbedingungen.
Die vierte Komponente des elektromagnetischen Feldes ist also der Lorentz-Eichterm? Der Lorentz-Eichungsterm wird zu Zwecken der Messgerätefixierung und zur Erfüllung der U(1)-Bedingungen auf Null gesetzt? Ich denke, das macht Sinn.
Ja, das ist eine faire Charakterisierung. Dieses skalare Feld ist normalerweise das Feld, das wir frei einstellen können, um das Messgerät zu fixieren. Wir können es frei wählen, weil sein zugehöriger Strom nicht in den Maxwell-Gleichungen auftaucht, also nur F gilt als körperlich. Ich würde dieses Skalarfeld nicht als zusätzliche Komponente des EM-Felds bezeichnen, aber ich nehme an, dass dies an dieser Stelle nur Semantik ist.
Ich habe ein wenig mehr Diskussion über die Befestigung von Messgeräten hinzugefügt und λ Auch.
Vielen Dank für die Bereitstellung des zusätzlichen Materials zu dieser Frage. Könnte dies mit den präsentierten Informationen nützlich sein, um die folgende Frage zu lösen: physical.stackexchange.com/questions/98513/… ?
In der GA-Community ist bekannt, dass Spin-1/2 der Algebra von Rotoren entspricht - dh Sie können Spin-1/2-Berechnungen genauso durchführen wie für die Multiplikation von Rotoren (oder in 3D Quaternionen). Ich stelle mir vor, dass Spin-1 nur den Drehungen von Vektoren entspricht, aber alles darüber hinaus habe ich keine Ahnung.

Dies ist eher ein erweiterter Kommentar, um die Kommentare zu Kyles Antwort anzusprechen

Zum Beispiel, wenn das elektrische und magnetische Feld eine Zeitkomponente hätte

In einem relativistischen Kontext sind die elektrischen und magnetischen Feldkomponenten keine Komponenten getrennter, verwandter Vektorfelder , sondern Komponenten eines Tensorfeldes 2. Rangs; die elektrischen und magnetischen Felder sind Teil eines geometrischen Objekts, nicht zwei.

Ein Hinweis darauf findet sich in der Tat in der Tatsache, dass das Magnetfeld in 3D eher ein Pseudo-Vektorfeld als ein Vektorfeld ist.

Die Frage „Was ist die Zeitkomponente des elektrischen und magnetischen Feldes“ setzt also tatsächlich eine Unwahrheit voraus ; es setzt voraus, dass die elektrischen und magnetischen Felder getrennte, aber verwandte Vierervektoren sind.

Aber das sind sie nicht .

Da ein Tensor des Ranges 2 zwei Indizes hat, können wir eigentlich von der Zeit-Zeit- Komponente, den Zeit-Raum- Komponenten und den Raum-Raum- Komponenten des Tensors sprechen, aber nicht von der Zeit oder den Raumkomponenten.

Ich konnte endlich die folgende Frage beantworten: physical.stackexchange.com/questions/103664/… Bitte gehen Sie dieser Frage nach, denn es gibt eine vierte Komponente im Feld.

Ziehen Sie den Faraday-Tensor und seinen Dual mit der 4-Geschwindigkeit zusammen. In Einheiten c 1 ,

E a = F a β U β

B a = F a β U β

wo F a β = 1 2   ε a β μ v   F μ v .

In einem lokalen Lorentz-Rahmen (mit c 1 ), kann die 4-Geschwindigkeit geschrieben werden U a = γ ( 1 , v ) , und die elektrischen und magnetischen 4-Vektoren können in bekannterer Notation geschrieben werden,

E a = γ (   v E , E + v × B   )

B a = γ (   v B , B v × E   )

Einheiten c = 1 . physical.stackexchange.com/users/84967/…
immer noch ... AFAIK, E ich = F 0 ich und B ich = ε ich j k F j k ...
Es ist richtig. Siehe z. B. Friedman & Stergioulas, "Rotating Relativistic Stars", Cambridge Monographs on Mathematical Physics, 2013, S. 129 - 134 physical.stackexchange.com/users/84967/…
In Ihrem obigen Kommentar in Einheiten c 1 , dann F 0 ich = E ich c . Außerdem, ε a β μ v ist eine Tensordichte; der entsprechende Levi-Civita-Tensor ist E a β μ v = 1 c   ε a β μ v . physical.stackexchange.com/users/84967/…
vergiss die Einheiten. Was ist v ? die Geschwindigkeit von was ?
Das U a = γ ( 1 , v ) ist die 4-Geschwindigkeit des Bezugsrahmens. Siehe z. B. Jackson, JD, „Classical Electrodynamics“, John Wiley & Sons Ltd., 1962, Kapitel 11. physical.stackexchange.com/users/84967/…
Hallo @user112613 : Tipp: Um einen Benutzer zu pingen, verwenden Sie @. Das Verlinken auf die Benutzerseite eines Benutzers wird ihn nicht anpingen.
@Qmechaniker Danke. Ich weiß noch nicht, wie genau dieser Stack-Tausch funktioniert. Ich bin über diese Frage gestolpert und dachte, ich könnte sie beantworten.
@ user112613 : Zur Kenntnis genommen.

Der Faraday 2 -Tensor F Die Beschreibung der elektromagnetischen Feldstärke ist antisymmetrisch, es ist also kein Platz darin 1 + 3 Abmessungen für alle zusätzlichen Komponenten außer der 3 elektrisch u 3 magnetische. Das ist einfach, aber warum ist es antisymmetrisch?

Ein Hinweis auf die Natur des elektromagnetischen Feldes kann gefunden werden, indem man die Maxwell-Gleichungen sowohl in Form des Faraday-Tensors ausdrückt F und der Maxwell-Tensor H :

[ ich F j k ] = 0 , [ ich H j k ] = J ich j k = def 1 3 ! ϵ ich j k l J l .
Physisch, F ( E , B ) ist die Feldstärke und H ( D , H ) ist die elektromagnetische Anregung. Ersteres gibt das Gaußsche Gesetz für den Magnetismus und das Faradaysche Induktionsgesetz an, während letzteres das Gaußsche Gesetz und das Ampère-Maxwell-Gesetz angibt.

Bemerkenswert an dieser Form ist, dass sich keine der Maxwell-Gleichungen überhaupt um die Metrik der Raumzeit kümmert. Vielmehr erscheint die Metrik als Teil des Hodge-Sterns in einem separaten Gesetz, das die Maxwell- und Faraday-Tensoren verbindet:

H F ,
Eine gute Art zu interpretieren ist, dass es sich um eine besonders einfache konstitutive Beziehung handelt , die die dielektrischen und magnetischen Eigenschaften der Raumzeit angibt. Beispielsweise können sowohl die nichtlineare Born-Infeld-Elektrodynamik durch eine alternative konstitutive Beziehung beschrieben werden, während die obigen Maxwell-Gleichungen gleich bleiben, als auch die Auswirkungen von QED-Vakuumkorrekturen erster Ordnung, die von Heisenberg und Euler abgeleitet wurden.

Wenn die drei elektrischen und magnetischen Vektorfelder aus dem vierkomponentigen Vierpotential stammen, gibt es dann eine vierte Komponente für das elektrische und magnetische Feld?

Lassen Sie uns nun die obige Beobachtung in ein Argument umwandeln. Elektromagnetismus ist nicht Gravitation, also brauchen wir vielleicht irgendwann die Metrik, um daraus eine vollständig vorhersagende Theorie zu machen, aber wir sollten in der Lage sein, die Gleichungen, die das elektromagnetische Feld selbst beschreiben, in eine Form zu bringen, die sowohl von der Metrik als auch von der Verbindung unabhängig ist. Daher sollten die Gleichungen des Elektromagnetismus auch dann noch Sinn machen, wenn die Raumzeit weder eine Metrik noch einen Zusammenhang hätte. Was neben der Topologie übrig bleibt, ist die differentielle Struktur .

Schlussfolgerung: Elektromagnetismus muss durch Differentialformen beschreibbar sein, und Differentialformen entsprechen kovarianten antisymmetrischen Tensoren. Im n Dimensionen, die Anzahl unabhängiger Komponenten von a k -Form ist C ( n , k ) . Wenn wir also wissen, dass sich die elektrischen und magnetischen Felder bei rahmenübergreifenden Transformationen vermischen und daher Teile desselben Tensors sein müssen / k -Form, die Gesamtzahl der Komponenten für n = 4 muss einer sein { 1 , 4 , 6 } .

Haben 4 denn sowohl das elektrische als auch das magnetische Feld würden machen 8 , also geht das nicht.


Übrigens, wenn wir auch schon wissen, dass die Feldstärke 2 -Form hat ein Potenzial, F = d EIN , dann EIN muss ein sein 1 -Form und haben vier unabhängige Komponenten ... aber wir hätten nicht erwarten sollen, dass all diese scheinbare Freiheit überhaupt physisch ist, weil EIN EIN + χ für alle 1 -bilden χ mit d χ = 0 produziert das gleiche F . Beachten Sie, dass F = d EIN impliziert, dass d F = 0 , das das Gaußsche Gesetz für den Magnetismus und das Faradaysche Induktionsgesetz ist.

Ich konnte endlich die folgende Frage beantworten: physical.stackexchange.com/q/103664 Bitte gehen Sie dieser Frage nach, da es eine vierte Komponente auf dem Gebiet gibt.

Die Antworten von Alfred Centauri und Kyle Kanos enthalten sachliche Aussagen, aber das Problem enthält mehr versteckte Geometrie. Es ist wahr, dass die elektrischen und magnetischen Feldvektoren keine echten Vektoren sind, sondern eher Pseudovektoren, wie von Alfred Centauri angegeben, die zu Feldtensoren des Ranges 2 gehören, wie von Kyle Kanos angegeben. Kyle Kanos erwähnte jedoch, dass die Felder aus der äußeren Berechnung stammen μ EIN v v EIN μ , was darauf hindeutet, dass die EM-Felder als mögliche Bivektoren des vierdimensionalen Raums verstanden werden können https://en.wikipedia.org/wiki/Bivector . Mit der Neuzuordnung eines Hodge Dual https://en.wikipedia.org/wiki/Hodge_dual kann man möglicherweise die Bivektorkomponenten auf vier Komponenten eines Vierervektors der Raumzeit neu abbilden, um die Notation zu verdichten. Dies würde eine mögliche vierte Komponente des EM-Feldes implizieren.

Der Bivektor-Link enthält eine Diskussion über den E&M-Feldtensor, die zeigt, dass die zeitähnliche Komponente das elektrische Feld ist, genau wie ich es gesagt hatte.
Das Hodge Dual in 4 Dimensionen bringt nur Bivektoren zu Bivektoren. In diesem Fall erhalten Sie nur den dualen EM-Feldtensor.
Ich habe eine zusätzliche Frage zu dieser Frage zu den beiden elektromagnetischen Tensoren gestellt: physical.stackexchange.com/q/103601